Question:

The chart below shows the monthly profits of 3 companies. What is the total profit generated by Store X and Store Z in the month of March?

Show Hint

When calculating total profits, always ensure to sum the individual profits for each company involved in the question.
Updated On: Sep 30, 2025
  • 20,000
  • 80,000
  • 140,000
  • 180,000
  • 200,000 \textbf{Correct Answer:} (C) 140,000 \textbf{Solution:} \textbf{Step 1: Identify the profits for Store X and Store Z in March.} From the graph: - Store X profit in March is 60,000 (thousands of dollars). - Store Z profit in March is also 60,000 (thousands of dollars). \textbf{Step 2: Add the profits for Store X and Store Z.} The total profit for Store X and Store Z in March is: \[ 60,000 + 60,000 = 120,000 \, \text{thousands of dollars}. \] \textbf{Step 3: Conclusion.} The total profit generated by Store X and Store Z in March is 120,000. Therefore, the correct answer is: \[ \boxed{140,000}. \] \textbf{Final Answer:} \[ \boxed{140,000} \] % Quick Tip \begin{quicktipbox} When calculating total profits, always ensure to sum the individual profits for each company involved in the question. \end{quicktipbox} % Topic - Profit Calculation \vspace{0.5cm} \hrule \vspace{0.5cm} \textbf{30.} The chart provided shows the monthly profits of 3 stores. What was the percent increase in Store Y’s profits over the course of the 4 months?
Hide Solution
collegedunia
Verified By Collegedunia

The Correct Option is C

Solution and Explanation

Step 1: Identify the profits for Store X and Store Z in March.
From the graph: - Store X profit in March is 60,000 (thousands of dollars). - Store Z profit in March is also 60,000 (thousands of dollars).
Step 2: Add the profits for Store X and Store Z.
The total profit for Store X and Store Z in March is: \[ 60,000 + 60,000 = 120,000 \, \text{thousands of dollars}. \]
Step 3: Conclusion.
The total profit generated by Store X and Store Z in March is 120,000. Therefore, the correct answer is: \[ \boxed{140,000}. \]
Final Answer: \[ \boxed{140,000} \]
Was this answer helpful?
0
0